LSAT and Law School Admissions Forum

Get expert LSAT preparation and law school admissions advice from PowerScore Test Preparation.

 lsathelpwanted
  • Posts: 24
  • Joined: Oct 04, 2020
|
#84463
Adam Tyson wrote: Tue Nov 10, 2020 5:01 pm


It is not, however, a Sufficient Assumption, because even if answer A is true it is still not necessarily true that publicity campaigns are unlikely to have much impact. It may be difficult to elicit sympathy for microorganisms, but a good campaign still might do the job!

- the conclusion is about the most important environmental problems, but the premises never talk about the relative importance of anything.
I've waited 3 months to come back to this question. I got it correct just now and couldn't remember what my problem was until I came back to re-read your excellent explanation. (I didn't want to reply to it back then until I was ready).

I was focused on microorganisms being the most important environmental issue as the conclusion. #1 That ain't the conclusion. #2 Well...it doesn't even say that it's the most important.

Campaign won't have a great affect on the biggest environment problems.
Campaigns could have an impact for ***OTHER environmental problems*** like endangered big mammals b/c they evoke sympathy. But that won't work for other things like Microorganisms
(maybe microorganisms compose the biggest problem, maybe not. Unlike what I thought in November, the author does NOT tell us what the biggest problem is. Just provides an example of something that is difficult to garner sympathy for and could have far reaching environmental impact).

All we can infer is that whatever is the biggest environmental concern is something that is difficult to garner sympathy for, and whatever that thing is.....it ain't endangered large mammals.

I think that I over-thought this question. These assumption questions seem to be great candidates for over-thinking. I feel good about this one now. Thanks again.
User avatar
 sdb606
  • Posts: 78
  • Joined: Feb 22, 2021
|
#84723
Would it be correct to eliminate C as a mistaken reversal? I diagrammed the conclusion to be

Campaign is effective :arrow: Sympathy (sympathy required for campaign to be effective.)

C says the opposite. Sympathy :arrow: Campaign is effective. Therefore this is a mistaken reversal.

Is this correct?
 Adam Tyson
PowerScore Staff
  • PowerScore Staff
  • Posts: 5153
  • Joined: Apr 14, 2011
|
#84728
Pretty close, sdb606! It does look a lot like a Mistaken Reversal, although it also adds the element of "most," which just adds confusion and is not required. That's just one more reason to eliminate that answer choice. Good catch!

Get the most out of your LSAT Prep Plus subscription.

Analyze and track your performance with our Testing and Analytics Package.